6

積分計算メモ(対数編)

484
1
$$$$

この記事は対数を含む積分によって得られた結果の羅列で、解説記事ではありません。
NKS 氏や 便利 氏らとのスペースで話題になったり、そこから少し派生したりした積分が中心です。導出過程については、別途気が向いた時に記事にしようかなと思います。

$\displaystyle\int_0^1x^a\ln^bx\,dx=(-1)^b\frac{b!}{(a+1)^{b+1}}$

$\displaystyle\int_0^nx^a\ln^bx\,dx=n^{a+1}\sum_{k=0}^b(-1)^k\binom{b}{k}\frac{k!}{(a+1)^{k+1}}\ln^{b-k}n$

$\displaystyle\int_0^1x^{n-1}\ln(1-x)=-\frac{H_n}{n}$

$\displaystyle\int_0^1x^{n-1}\ln^2(1-x)=\frac{H^2_n+H_n^{(2)}}{n}$

$\displaystyle\int_0^1x^{n-1}\ln^3(1-x)=-\frac{H^3_n+3H_nH_n^{(2)}+2H_n^{(3)}}{n}$

$\displaystyle H_n^{(m)}=\sum_{k=1}^n\frac{1}{k^m}=\frac{1}{1^m}+\frac{1}{2^m}+\cdots+\frac{1}{n^m}$
$\displaystyle\int_0^1x^{n-1}\ln^m(1-x)dx=\frac{(-1)^mm!}{n}\cdot h_m\left(1,\frac{1}{2},\cdots,\frac{1}{n}\right)$

$h_k(x_1,\cdots,x_n)$ Complete homogeneous symmetric polynomial を表し、

$\displaystyle h_k(x_1,x_2,\cdots,x_n)=\sum_{1\leq i_1\leq\cdots\leq i_k\leq n}x_{i_1}x_{i_2}\cdots x_{i_k}$

$\displaystyle\int_0^1\frac{\ln^2(1+x)}{x}dx=\frac{1}{4}\zeta(3)$

$\displaystyle\int_0^1\frac{\ln^3(1+x)}{x}dx=-\frac{21}{4}\ln2\zeta(3)+\frac{\pi^2\ln^22}{4}-\frac{\ln^42}{4}+\frac{\pi^4}{15}-6\mathrm{Li}_4\left(\frac{1}{2}\right)$

$\displaystyle\int_0^1\frac{\ln^2(1-x)}{x}dx=2\zeta(3)$

$\displaystyle\int_0^1\frac{\ln^3(1-x)}{x}dx=-\frac{\pi^4}{15}$

$\displaystyle\int_0^1\frac{\ln^n(1+x)}{x}dx=n!\zeta(n+1)-\sum_{k=0}^n k!{n\choose k}\ln^{n-k}(2)\operatorname{Li}_{k+1}\left(\frac12\right)+\frac{\ln^{n+1}2}{n+1}$


$\displaystyle\int_0^1\frac{\ln^n(1-x)}{x}dx=(-1)^nn!\zeta(n+1)$
$\displaystyle\int_0^x\frac{\ln^2(1+t)}{t}dt=\ln x\ln^2(1+x)-\frac{2}{3}\ln^3(1+x)-2\ln(1+x)\mathrm{Li}_2\left(\frac{1}{1+x}\right)-2\mathrm{Li}_3\left(\frac{1}{1+x}\right)+2\zeta(3)$

$\displaystyle\int_0^x\frac{\ln^2(1-t)}{t}dt=\ln x\ln^2(1-x)+2\ln(1-x)\mathrm{Li}_2(1-x)-2\mathrm{Li}_3(1-x)+2\zeta(3)$

$\displaystyle\int_0^1 \ln(1+x)\ln(1-x)=2\ln^2 2-2\ln 2-\dfrac{\pi^2}{6}$

$\displaystyle\int_0^1\ln^2(1+x)\ln^2(1-x)dx=-8(\zeta(2)+\zeta(3)-3)-\zeta(4)+4(2\ln2\zeta(2)+\ln^2\zeta(2)+2\ln2\zeta(3)-6\ln2+3\ln^22-\ln^32)+\ln^42$

$\displaystyle\int_0^1\ln^n(1+x)\ln^n(1-x)dx=\frac{1}{4}\lim_{\substack{x\to 1\\ y\to 1}}\frac{\partial^{2n}}{\partial x^n\partial y^n}(2^{x+y}B(x,y))$
$\displaystyle B(x,y)=\int_0^1u^{x-1}(1-u)^{y-1}du=\frac{\Gamma(x)\Gamma(y)}{\Gamma(x+y)}$

$\displaystyle\int_0^1\frac{\ln(1+x)\ln(1-x)}{x}dx=-\frac{5}{8}\zeta(3)$

$\displaystyle\int_0^1 \frac{\ln^2(1+x)\ln(1-x)}{x}dx=-\frac{\pi^4}{240}$

$\displaystyle\int_0^1 \frac{\ln(1+x)\ln^2(1-x)}{x}dx=-\frac{5}{8}\zeta(4)+2 \left( \operatorname{Li_4}\left(\frac12\right)+\frac78\ln2\zeta(3)-\frac14\ln^22\zeta(2)+\frac{1}{24} \ln^24 \right)$

$\displaystyle\int_0^1\frac{\ln^2(1+x)\ln^2(1-x)}{x}dx=\frac{2\ln^52}{15}-\frac{2\ln^32\zeta(2)}{3}+\frac{7\ln^22\zeta(3)}{4}-\frac{25\zeta(5)}{8}+4\ln2\mathrm{Li}_4\left(\frac{1}{2}\right)+4\mathrm{Li}_5\left(\frac{1}{2}\right)$

$\displaystyle\int_0^1\frac{\ln^3(1+x)\ln(1-x)}{x}dx=-6\mathrm{Li}_5\left(\frac{1}{2}\right)-6\ln2\mathrm{Li}_4\left(\frac{1}{2}\right)+\frac{3}{4}\zeta(5)+\frac{21}{8}\zeta(2)\zeta(3)-\frac{21}8\ln^22\zeta(3)+\ln^32\zeta(2)-\frac15\ln^52$

$\displaystyle\int_0^1\frac{\ln(1+x)\ln^3(1-x)}{x}dx=6\mathrm{Li}_5\left(\frac{1}{2}\right)+6\ln2\mathrm{Li}_4\left(\frac{1}{2}\right)-\frac{81}{16}\zeta(5)-\frac{21}{8}\zeta(2)\zeta(3)+\frac{21}{8}\ln^22\zeta(3)-\ln^32\zeta(2)+\frac{1}{5}\ln^52$

$\displaystyle\int_0^1 \frac{\ln^n(1+x)\ln(1-x)}{x}dx=\,?$
$\displaystyle\int_0^1 \frac{\ln(1+x)\ln^n(1-x)}{x}dx=(-1)^n n! \sum_{k=1}^\infty\frac{H_k^{(n+1)}}{k\cdot2^k}$

$\displaystyle\int_0^1 \frac{\ln(1+x)\ln(1-x)}{1+x}dx = \frac{\ln^32}{3}-\frac{\pi^2\ln2}{12}+\frac{\zeta(3)}{8}$

$\displaystyle\int_0^1\frac{\ln^2(1+x)\ln^2(1-x)}{1+x}dx=\frac{63\zeta(5)}{8}-\frac{9\ln2\zeta(4)}{2}+4\ln^22\zeta(3)-\frac{4\ln^32\zeta(2)}{3}-2\zeta(2)\zeta(3)+\frac{7\ln^52}{30}-4\mathrm{Li}_5\left(\frac{1}{2}\right)$

$\displaystyle\int_0^1\frac{\ln(1+x)\ln(1-x)}{1+x^2}\ dx=\text{Im}\left(\operatorname{Li}_3(1+i)\right)-\frac{\pi^3}{32}-G\ln2$

$\displaystyle\int_0^1\frac{\ln(1+x)\ln^2x\ln{(1-x)}}{x}dx=\frac{3\zeta(2)\zeta(3)}{4}-\frac{27\zeta(5)}{16}$

$\displaystyle\int_0^1\frac{\ln(1+x)\ln^4x\ln{(1-x)}}{x}dx=\frac{9\zeta(3)\zeta(4)+45\zeta(2)\zeta(5)}{4}-\frac{363\zeta(7)}{16}$

$\displaystyle \int_0^1\frac{\ln^p(1+x)\ln^qx\ln^r{(1-x)}}{x}dx=p!q!r!\left(-1\right)^{q}\sum_{n\geq1}\sum_{k\geq1}\frac{\left(-1\right)^{n}H_{n-1}^\left(\overbrace{1,\dots,1}^{\scriptstyle r-1}\right)H_{k-1}^\left(\overbrace{1,\dots,1}^{\scriptstyle p-1}\right)}{nk\left(n+k\right)^{q+1}}$
$\displaystyle H_{n}^{\left(m_{1},\dots,m_{k-1}\right)}=\sum_{1\leq n_{k-1}< n_{k-2}<\dots< n_{1}\leq n}\frac{1}{n_{1}^{m_{1}}\cdots n_{k-1}^{m_{k-1}}}$
投稿日:77
更新日:77

この記事を高評価した人

高評価したユーザはいません

この記事に送られたバッジ

バッジはありません。

投稿者

紗夜
紗夜
13
890

コメント

他の人のコメント

コメントはありません。
読み込み中...
読み込み中